Themis- Missed PQs (Contracts & Family Law)

Ace your homework & exams now with Quizwiz!

1) A party who substantially performs contractual obligations (i.e., commits a minor breach) _________ recover on the contract even though that party has not rendered full performance. The substantially performing party can generally recover the _______________ minus ____________________. 2) In contrast, a party who commits a material breach by failing to substantially perform __________ recover under the contract. The breaching party can only recover in _________________ for any benefit conferred on the nonbreaching party minus damages for the breach.

1) can contract price any cost that the nonbreaching party incurred to receive full performance 2) cannot restitution

A purchase-money mortgage (PMM) is often used by buyers who don't have enough money for a traditional down-payment or who cannot get a large enough bank mortgage. The PMM funds must be used to purchase the property and are given either to the seller as part of the purchase price or to a third party lender. In the event that a mortgagor-borrower defaults, who has priority if there is a seller PMM and third party lender PMMs?

A seller's PMM has priority over a third-party's PMM. Priority of third party PMMs against each other are determined chronologically (subject to applicable exceptions).

Regarding the waiver of an express condition, which of the following statements is FALSE? A. An express condition can be waived only by express language. B. A party may waive an express condition by hindering its occurrence. C. A party may waive an express condition by wrongfully interfering with its occurrence. D. When the occurrence of an express condition depends on the actions of a party, the actions undertaken by the party are judged by a good-faith standard.

A. An express condition can be waived only by express language.

A gardener and a carpenter contracted in writing for the carpenter to repair the gardener's four identical beehives for $500 each. The contract was signed by both parties and provided that the gardener would pay the carpenter $2,000 upon delivery of the fourth repaired beehive. The gardener immediately delivered all four beehives to the carpenter for repair. The carpenter repaired and delivered the first two beehives without any problems and without demanding payment. However, upon delivery of the third repaired beehive, the carpenter demanded a payment of $1,500. Is the gardener required to make the demanded payment at this time? A. No, because she has no duty to pay the carpenter anything until the last beehive is repaired and delivered. B. No, because the course of performance between the parties has established that payment is not due upon the delivery of each repaired beehive. C. Yes, because the carpenter is entitled to the fair value conferred on the gardener. D. Yes, because the contract is divisible with respect to the repair and delivery of each beehive.

A. No, because she has no duty to pay the carpenter anything until the last beehive is repaired and delivered. Why? *Although courts prefer to interpret contracts as divisible, they will not do so in contradiction of the contract's express terms—e.g., when the contract expressly states that it is indivisible.

On August 1, a buyer and a seller contracted in writing for the sale of the seller's duck farm. The contract provided that the closing would occur on September 15. On September 1, the buyer sent the seller an email containing the following: "After discussing our August 1 contract with my financial advisors and some experts in the field, I am increasingly concerned that entering the duck market at this time is a grave mistake. As such, I do not intend to buy your duck farm unless I am legally obligated to do so." If the seller sues the buyer for breach of contract on September 1, is the seller likely to succeed? A. No, because the buyer's statement is neither a present breach nor a repudiation of the August 1 contract. B. No, because the seller has not yet accepted or relied upon the buyer's repudiation. C. Yes, because the buyer has committed a breach by anticipatory repudiation. D. Yes, because the buyer's statement created reasonable grounds for the seller's insecurity with respect to the buyer's performance.

A. No, because the buyer's statement is neither a present breach nor a repudiation of the August 1 contract. Why? *A contracting party must generally wait until performance is due before suing the other party for breach of contract. But a party can sue immediately under the doctrine of anticipatory repudiation if, before performance is due, the other party clearly and unequivocally indicates by words or conduct that it cannot or will not perform.

During the warm months of the year, the owner of a fur coat stored it with the furrier from whom she had bought it. While the coat was at the furrier's store, a salesperson, mistakenly thinking that the coat was for sale, sold it to a customer. The customer was allowed to reduce the purchase price by the amount of an outstanding debt owed by the furrier to the customer; the customer paid the remainder in cash. In the process of purchasing the coat, the customer was told by the salesperson about the furrier's storage service but, like the salesperson, was unaware that the coat was not part of the store's merchandise. After the sale, the owner learned of the transaction between the furrier and the customer. Since the coat had significant sentimental value to the owner, she sought its return from the customer. When the customer refused, the owner filed an action to recover the coat from the customer. Will the owner likely prevail? A. No, because the customer was a good-faith purchaser of the coat that had been entrusted to the furrier. B. No, because the owner is entitled to damages from the furrier. C. Yes, because the customer did not give full value in acquiring the coat. D. Yes, because the furrier transferred only voidable title in the coat to the customer.

A. No, because the customer was a good-faith purchaser of the coat that had been entrusted to the furrier. Why? *A merchant entrusted with goods has the power to convey good title to a buyer in the ordinary course. A buyer in the ordinary course is someone who buys goods (1) in good faith, (2) without knowledge that the sale violates the owner's rights to the goods, and (3) from a merchant in the business of selling goods of that kind.

A wheat farmer contacted an agricultural services company in May to inquire about hiring workers for a five-day period toward the beginning of the summer-long harvest season to assist the farmer in harvesting his wheat crop. After some negotiations, the farmer entered into a written contract with the company "to provide five workers for a five-day period starting in the first week of June for a cost of $5,000." On May 31, the company's workers went on strike. On June 9, the strike ended, and the company's workers began harvesting wheat on the farmer's farm for the next five days. The farmer subsequently refused to pay the company, claiming that the company's delay in performance excused his obligation to pay. Is the farmer's obligation to pay excused? A. No, because the delay did not deprive the farmer of the substantial benefit of the bargain. B. Yes, because starting in the first week of June was an express condition of the contract. C. Yes, because substantial performance does not excuse a breach for commercial contracts. D. Yes, because the delay was a material breach as the harvesting season had already begun.

A. No, because the delay did not deprive the farmer of the substantial benefit of the bargain. Why? *Under common law, a material breach of contract allows the nonbreaching party to withhold performance. A breach is material when the nonbreaching party does not receive the substantial benefit of its bargain, so substantial performance does not constitute a material breach (except as to an express condition).

The owner of a ferry boat operated the boat only during daylight hours during the summer months of June, July, and August. On March 1, the owner entered into a written agreement with a man to serve as the captain of the boat for the upcoming season. On May 1, the owner contracted with a woman to serve as the captain of the boat. On May 30, the man was diagnosed with an illness, and the treatment for this illness prevented him from being employed until the following year. On May 31, the owner learned of the man's illness and told the man not to worry about their contract as he had found someone else to serve as captain of the boat. The woman served as captain of the boat for the summer months of June, July, and August that year. On September 1, the man sued the owner for damages based on a breach of their contract. Can the man recover damages based on breach of contract? A. No, because the man was unable to serve as the captain of the boat during the summer months. B. No, because the owner informed the man about the owner's contract with the woman prior to June 1. C. Yes, because the owner did not inform the man of the owner's contract with the woman until after the owner learned of the man's illness. D. Yes, because the owner's contract with the woman constituted an anticipatory breach of the owner's contract with the man.

A. No, because the man was unable to serve as the captain of the boat during the summer months. Why? *A nonrepudiating party who materially breaches the contract cannot recover damages for the other party's anticipatory breach because the material breach discharges the other party's duty to perform.

A plastics manufacturer saw an advertisement for a plastic extruding machine. The manufacturer contacted the seller, who was a merchant of plastic extruding machines, and made arrangements to inspect the machine at the seller's place of business. The manufacturer walked around the machine once and stated: "Yes, this looks like what I need." When the manufacturer asked the price, the seller stated a price that was less than half the amount a similar, functioning, used machine commanded on the market. The manufacturer was surprised at the low price but did not inquire as to the reason. The seller encouraged the manufacturer to perform a closer inspection before finalizing the purchase and offered to open the motor housing so that the motor could be examined, but the manufacturer declined. The parties completed the sale. The manufacturer transported the machine to his factory. When it arrived, he first learned that the motor was burned out and required complete replacement, as would have been readily apparent upon visual inspection had the motor housing been opened. Replacing the motor would cost roughly the amount that the manufacturer had paid for the machine. The manufacturer contacted the seller to return the machine, but the seller refused to take it back. The manufacturer then filed suit against the seller. Will the manufacturer likely prevail? A. No, because the manufacturer waived any implied warranties by failing to inspect the machine. B. No, because the seller made no claims regarding the operability of the machine. C. Yes, because the manufacturer's unilateral mistake regarding the condition of the machine was caused by the seller. D. Yes, because the seller violated the implied warranty of merchantability by selling a machine with a burned-out motor.

A. No, because the manufacturer waived any implied warranties by failing to inspect the machine. Why? *The implied warranty of merchantability requires a merchant-seller to provide a buyer with merchantable goods. This warranty may be disclaimed for defects that an examination would have revealed if, before entering the contract, the buyer examined the goods as fully as desired or refused to examine them.

While attending a rodeo on August 20, a hatmaker entered into a valid, written agreement with the rodeo manager to make 500 leather cowboy hats for an upcoming rodeo event at a price of $75 per hat. Per the agreement, the rodeo manager agreed to pay one-fourth of the total purchase price to a tannery owner to whom the hatmaker owed a debt for a previous leather order. The hatmaker and the rodeo manager made no mention of the agreement to the tannery owner. On August 25, the hatmaker changed his mind about paying one-fourth of the purchase price to the tannery owner. The hatmaker and the rodeo manager subsequently executed a valid modification of the original agreement. The rodeo manager's brother had also been present on August 20 when the original agreement was executed, but he did not know about the August 25 modification of the agreement to no longer pay the tannery owner. On August 30, the brother, who was friends with the tannery owner, called and told the tannery owner that his debt from the hatmaker would finally be paid off. However, the rodeo manager refused to pay one-fourth of the purchase price to the tannery owner. If the tannery owner sues the rodeo manager for one-fourth of the purchase price, will he likely recover? A. No, because the tannery owner did not rely on the August 20 agreement between the hatmaker and the rodeo manager. B. No, because there was no consideration for the promise to pay the tannery owner by the hatmaker and the rodeo manager. C. Yes, because the tannery owner had the right to sue the rodeo manager to enforce the contract between the rodeo manager and the hatmaker. D. Yes, because the rodeo manager agreed to pay one-fourth of the purchase price to the tannery owner on August 20.

A. No, because the tannery owner did not rely on the August 20 agreement between the hatmaker and the rodeo manager.

A college student wanted to purchase a car so that he could visit friends at other nearby colleges. After looking at a few different cars, the student found one he liked and entered into negotiations with a salesman for the purchase of the car. The student, who was 19 years old, chose to finance the car through the dealership and negotiated with the salesman on the terms and monthly payments of the car loan. However, because the student had no credit history, the dealership would not finance the purchase unless the student was able to provide a surety for the loan. The student contacted his wealthy uncle, who agreed to guarantee the car loan for the student. The uncle accompanied the student to the car dealership and met with the salesman. The uncle stated, "I hereby guarantee the loan so that my nephew can buy this car and enjoy it." The salesman and the uncle shook hands, and the dealership provided the student with the loan necessary to purchase the car. The student took possession of the car but failed to make any payments on the loan. As a result, the entire amount has become due. Is the dealership likely to prevail in an action against the uncle to recover the entire amount of the loan? A. No, because the uncle's agreement to guarantee the loan was not in writing. B. No, because there was no consideration for the uncle's promise to pay the debt. C. Yes, because the uncle agreed to guarantee the loan. D. Yes, because the uncle is primarily liable on the loan.

A. No, because the uncle's agreement to guarantee the loan was not in writing. Why? *Suretyships fall within the statute of frauds, so they must generally be in writing and signed by the party against whom enforcement is sought to be enforceable. However, a suretyship is enforceable without a writing if it was made mainly for the surety's economic advantage (not the principal's benefit) or to indemnify the creditor.

A carpenter and a neighbor agreed that the carpenter would construct a tree fort for the neighbor's child for $1,200 to be paid upon completion of the fort. When the carpenter was halfway through the project, he abandoned the tree fort to work on far more profitable projects. At that time, he had spent $300 on materials. The neighbor found a handyman who completed the tree fort for $500. When the carpenter approached the neighbor for payment, the neighbor refused. If the carpenter sues the neighbor for breach, what is the likely amount of damages that the carpenter will be awarded under the contract? A. Nothing B. $300 C. $600 D. $700

A. Nothing Why? *The neighbor's payment of $1,200 to the carpenter was conditioned upon the carpenter's building a tree fort, and the carpenter's building a tree fort was conditioned upon the neighbor's payment of $1,200. As a contract for the performance of services, rather than for the sale of goods, the carpenter's substantial compliance with his obligation would have been sufficient to require the neighbor to pay him, although the neighbor would have been entitled to damages to the extent that the carpenter failed to fully complete the task. Here, however, the carpenter finished only one-half of the tree fort, and thus failed to substantially complete his contractual obligation. Consequently, the neighbor does not owe the carpenter any damages based on his breach of the contract. Answer choice B is incorrect because, as the breaching party, the carpenter is not entitled to recover his reliance damages of $300.

Which of the following is FALSE with regard to child support? A. Support obligations are based on the parents' ability to pay. B. Both parents are legally required to support their minor children. C. The right to support belongs to the child. D. Parents may not enter into an agreement to limit the amount of child support.

A. Support obligations are based on the parents' ability to pay.

In late March, a jeweler contracted to create and sell a ruby ring to a buyer on May 1. In early April, the price of rubies rose by 25 percent, and the jeweler could not purchase a ruby without losing money on the transaction. However, the jeweler had an emerald that had been purchased for another customer who had canceled the order. The jeweler created the ring for the buyer using the emerald. On May 1, the jeweler tendered the emerald ring to the buyer in good faith, explaining the situation and pointing out that the emerald ring was the same value that the ruby ring would have been. The buyer refused to accept the emerald ring and refused to pay the jeweler the contract price. Which of the following most accurately describes the legal relationship between the jeweler and the buyer? A. The buyer can immediately bring a valid breach of contract claim against the jeweler for his failure to tender a ruby ring. B. The buyer must give the jeweler an opportunity to cure the breach before bringing a claim against him. C. The jeweler can immediately bring a valid breach of contract claim against the buyer for rejecting the emerald ring. D. The jeweler's duty under its contract with the buyer was excused by the rise in the price of rubies.

A. The buyer can immediately bring a valid breach of contract claim against the jeweler for his failure to tender a ruby ring. Why? *Under the UCC, a seller must tender goods conforming to the contract to the buyer. The UCC requires "perfect tender," and substantial performance will not suffice except for installment contracts, or when the parties have agreed otherwise. Thus, under the UCC's perfect tender rule, the jeweler has breached the contract with the buyer, and the buyer can immediately bring an action against the jeweler.

On May 10, the coach of a youth-league baseball team sent a letter to a supplier asking the supplier to promptly ship 20 red jerseys to the coach. On May 15, the supplier received this letter and sent the coach a reply letter accepting the offer. On May 16, the supplier realized that he had no red jerseys with which to fill the order and sent the coach 20 blue jerseys with a note that the blue jerseys were tendered as an accommodation. The coach received the jerseys and accommodation note on May 18 and received the supplier's acceptance letter on May 19. On May 20, which of the following is a correct statement of the parties' legal rights and duties? A. The coach can either accept or reject the blue jerseys and, in either event, recover damages, if any, for breach of contract. B. The coach can either accept or reject the blue jerseys, but if he rejects them, he will thereby waive any remedy for breach of contract. C. The supplier's shipment of nonconforming goods constituted an acceptance of the coach's offer, thereby creating a contract for the sale of the blue jerseys. D. The supplier's shipment of the blue jerseys constituted a counteroffer.

A. The coach can either accept or reject the blue jerseys and, in either event, recover damages, if any, for breach of contract.

A professor listed her car for sale on an online site. A buyer called and asked a series of questions, including the following, "I am not much of a music lover, but how does the stereo sound?" The professor replied that it sounded great. In reality, the stereo had short-circuited months earlier and blown out all the speakers in the process. Based on the conversation, the buyer entered into a written contract with the professor to purchase the car at a specified price. The contract provided for the buyer to pay for the car and take possession of it within three days. The next day, remorseful over her lie, the professor had the stereo and speakers restored to factory condition. On the day the buyer was supposed to take possession of the car, he learned from a student that the professor had lied about the condition of the stereo, and decided not to pay for the car. When the professor called the buyer later that day, the buyer told her he was not purchasing the car. The professor admitted her lie, and told the buyer that the stereo and speakers had been replaced. Nevertheless, the buyer refused to pay for the car. In a subsequent breach of contract suit, who will prevail? A. The professor, because she repaired the stereo so that it conformed to the buyer's expectations. B. The professor, because the condition of the stereo was not material to the buyer. C. The buyer, because the professor acted in bad faith in inducing the buyer to enter into the contract. D. The buyer, because he had the right to terminate the contract following the professor's misrepresentation.

A. The professor, because she repaired the stereo so that it conformed to the buyer's expectations. Why? *A misrepresentation is an untrue assertion of fact. Fraudulent misrepresentation requires proof of the following: (i) the misrepresentation is made knowingly and with intent to mislead the other party (i.e., it is fraudulent), (ii) the misrepresentation induced assent to the contract, and (iii) the adversely affected party justifiably relied on the misrepresentation. A fraudulent misrepresentation need not be material, and may make the contract voidable at the adversely affected party's option. The contract is no longer voidable, however, if, following a misrepresentation but before the deceived party has avoided the contract, the facts are cured so as to be in accord with the facts that were previously misrepresented.

An auto manufacturer contracted with a supplier to provide speakers for 10,000 vehicles at a total price of $600,000. Prior to the date fixed for delivery of the speakers, the supplier, without justification, informed the manufacturer that it could not supply the speakers. The manufacturer immediately sought quotes from other suppliers. The manufacturer received a quote from a second supplier, who had previously provided the manufacturer with speakers in a timely and satisfactory manner, to sell 10,000 speakers for $800,000. A short time later, the manufacturer received an offer from a third supplier, with whom the manufacturer had not previously worked: to supply the 10,000 speakers for $600,000. The manufacturer, reasonably concerned that the third supplier would be unable to provide the speakers in a timely and satisfactory manner, entered into a contract with the second supplier. Subsequently, the first supplier told the manufacturer that it would be able to supply the speakers by the original delivery date and, although not requested by the manufacturer, provided the manufacturer with adequate evidence that the speakers would be timely delivered. The manufacturer told the first supplier not to deliver the goods and instead acquired them from the second supplier. The auto manufacturer sued the first supplier for $200,000. Should the auto manufacturer prevail? A. Yes, because the auto manufacturer is entitled to recover the cost of acquiring substitute speakers. B. Yes, because the first supplier had repudiated its contractual obligation. C. No, because the auto manufacturer was required to mitigate its damages by accepting the third supplier's lower offer. D. No, because the first supplier withdrew its repudiation prior to the time for performance.

A. Yes, because the auto manufacturer is entitled to recover the cost of acquiring substitute speakers. Why? *A buyer can seek replacement goods and can recover the difference between the contract price and the cost of the replacement goods (i.e., cover). Here, the auto manufacturer paid $800,000 to acquire the speakers that the first supplier was contractually obligated to deliver for $600,000. Consequently, the manufacturer could recover $200,000 from the first supplier.

FILL IN THE BLANK. When both parties agree that another party will assume the contractual obligations of one of the parties, a __________ has occurred. A. novation B. delegation C. rescission D. cancellation

A. novation

On April 1, a buyer agreed in writing to purchase an antique car from a seller for $20,000. The parties met on April 10, the scheduled date of the sale, at which time the buyer accepted the car and gave the seller a check for $15,000. The buyer, seeking to create an accord and satisfaction, had added the following conspicuous notation on the check: "This check is in full and final satisfaction of my obligation under our April 1 agreement." The seller did not realize that the check was for only $15,000 and that it contained the notation until the seller sought to deposit it at her bank later that day. Needing the money, the seller deposited the check anyway. If the seller sues the buyer for breach of contract seeking damages of $5,000, the difference between the amount paid and the contract price, will the buyer's accord and satisfaction defense likely succeed? A. No, because the buyer could not modify the agreement without consideration. B. No, because the buyer did not dispute the initial purchase price of the car. C. Yes, because the notation on the check formed a substituted contract. D. Yes, because the seller deposited the check knowing it was offered in full and final satisfaction of the buyer's obligation.

B. No, because the buyer did not dispute the initial purchase price of the car. Why? *If a debt is disputed in good faith, then the debtor can offer to satisfy the debt by giving the creditor a check with a conspicuous "payment in full" notation. But if the debt is certain and undisputed, then it cannot be satisfied by a check for a lesser amount—even if the creditor cashes the check.

Prior to her death, a celebrity commissioned an artist to paint a portrait of her. The celebrity hired this particular artist because he painted using an old-fashioned and rarely used style that required two months of daily appointments during which the subject would sit for a few hours each day. The contract between the parties specified that this live-model method would be used and that the celebrity would deliver increasing payments throughout the process, with the first payment occurring after two weeks of painting. One week into the process, after the painting had begun, the celebrity died. Her family demanded that the artist continue with the painting, using photographs as a substitute for the daily sessions. Is the artist required to complete a painting of the celebrity? A. No, because no payment had yet occurred. B. No, because the celebrity died after only one week. C. Yes, because the artist can complete the painting by relying on photos of the celebrity. D. Yes, because the artist had already begun painting the celebrity.

B. No, because the celebrity died after only one week. Why? *A contracting party's duty to perform is discharged by impracticability when (1) an unanticipated or extraordinary event makes it impracticable for the party to perform, (2) the contract was formed under a basic assumption that the event would not occur, and (3) the party seeking discharge was not at fault in causing the event to occur.

As part of a divorce settlement, an ex-husband purchased an annuity from an insurance company to be paid to his ex-wife so that she would receive a fixed amount quarterly for the duration of her life. Within a week after the purchase, the ex-wife learned that she had a fatal illness, which had not previously manifested itself but had existed for some time. She died two months later, prior to receiving any payments from the annuity. The ex-husband has filed suit to rescind the annuity contract. Will the ex-husband be likely to prevail? A. No, because the annuity contract was a third-party beneficiary contract. B. No, because the ex-husband assumed the risk of his ex-wife's death. C. Yes, because the ex-wife's death frustrated the purpose of the annuity. D. Yes, because the ex-husband and the insurance company made a mutual mistake as to the ex-wife's health.

B. No, because the ex-husband assumed the risk of his ex-wife's death. Why? *A mutual mistake may render a contract voidable by the adversely affected party. But a party assumes the risk of the mistake—and cannot void the contract—if the party knew at the time of the contract that he/she had limited knowledge of the facts and accepted this knowledge as sufficient.

The owner of a restaurant who highlighted local ingredients when creating his menu bought cheese and other dairy products from a local dairy farmer. The owner and the farmer had entered into written requirements contracts each spring for the past 10 years. In the winter of the tenth year, the farmer purchased a substantial amount of new dairy cows and expanded his farming capabilities. He notified all customers that he would have a higher volume and amount of available products the following spring and would adjust deliveries accordingly. The owner responded with a date he wished the products to be delivered, as per custom, but said nothing else. On the agreed-upon date, the farmer delivered substantially more products than he had customarily provided. The owner attempted to accept half of the shipment, as that was roughly his customary quantity, but the farmer stated that the products were already packaged and that the owner should have spoken up after receiving the notice from the farmer. The owner then rejected the shipment in its entirety. Did the owner breach the contract with the farmer as to this shipment? A. No, because no contract existed, as the parties did not agree to a quantity. B. No, because the farmer made a nonconforming tender of goods. C. Yes, because the owner should have given the farmer time to cure the nonconformity. D. Yes, because the owner rejected the shipment in its entirety.

B. No, because the farmer made a nonconforming tender of goods.

A wholesaler and an orange grower entered into a valid contract for 25 crates of Grade 1 oranges, to be delivered on or before the end of the month. On the last day of the month, the grower delivered 25 crates of Grade 2 oranges to the wholesaler. Based on the price and popularity of Grade 2 oranges in the area, the grower reasonably believed that the wholesaler would accept these oranges at a reduced price. When the wholesaler refused to accept the delivery, the grower promised to return the following morning with 25 crates of Grade 1 oranges. When the farmer did, the wholesaler again refused to accept the oranges. Is the wholesaler's rejection of the Grade 1 oranges proper? A. No, because the earlier rejection of the Grade 2 oranges at a reduced price was improper. B. No, because the grower cured the nonconforming tender. C. Yes, because the grower's delivery of the Grade 1 oranges was untimely. D. Yes, because the perfect tender rule applies to sales of goods.

B. No, because the grower cured the nonconforming tender. Why? *Although the perfect tender rule generally applies to a sale of goods, the seller has a right to cure a defective tender—even after the time for performance under the contract has elapsed—if the seller had reasonable grounds to believe that the buyer would accept the goods despite the nonconformity. In curing the defect, the seller must notify the buyer of its intent to do so and effect the cure within a reasonable time. Although the grower delivered the Grade 1 oranges to the store after the time for performance had passed and thus failed to satisfy the perfect tender rule, because the grower had reasonable grounds to believe that the wholesaler would accept the Grade 2 oranges at a reduced price, the grower had a reasonable time to correct the defective tender. Because the grower delivered the conforming oranges the following day, it is likely that the wholesaler's rejection of the oranges was improper.

A buyer at a local market offered to purchase a large mirror from an artist for $1,000. The artist stated that he wanted to wait to see how many people went through the market that day before he decided on whether he would accept the offer. The buyer agreed to wait until the next morning for the artist's decision. The next morning, the buyer returned to the market only to learn that the mirror had been dropped and shattered. The buyer believed that the destruction of the mirror terminated his original offer, but because the frame of the mirror was still in good condition, the buyer decided to buy the frame instead. The buyer wrote a check for $500 and gave it to the artist without further remark. The artist loaded the empty frame into the buyer's vehicle and, believing that he had accepted the buyer's original offer, demanded the remaining $500 the buyer had offered the day before. Is the buyer liable for the remaining $500? A. No, because the buyer believed that the original offer had terminated. B. No, because the original offer terminated. C. Yes, because the artist thought that he had accepted the original offer. D. Yes, because the original offer was still valid.

B. No, because the original offer terminated. Why? *An offer can be terminated by operation of law—e.g., when the subject matter of the offer is destroyed.

A mechanic and a farmer contracted in writing for the repair of the farmer's tractor, with a payment of $2,000 due upon completion. The mechanic called the farmer on April 15 to inform him that the work was complete. When the farmer went to pick up the tractor the next day, he told the mechanic that due to an unforeseen rise in feed costs, he could not pay the full contract price. The farmer paid the mechanic $1,000. The mechanic told the farmer that, if the farmer promised to pay the remainder by June 1, then the mechanic would not sue to recover the remaining $1,000. The farmer orally agreed. On May 1, the mechanic sued the farmer for the unpaid $1,000, and the farmer filed a motion to dismiss. Should the court grant the motion to dismiss? A. No, because the new cost of feed is not an unforeseen difficulty that would allow for modification of the existing contract. B. No, because there is no consideration to support the mechanic's promise not to sue. C. Yes, because a promise to allow a debtor to delay payment on a past debt is enforceable without consideration. D. Yes, because the payment of $1,000 was sufficient consideration to support the mechanic's promise not to sue.

B. No, because there is no consideration to support the mechanic's promise not to sue. Why? *Under the preexisting-duty rule, a promise to perform a duty that a party is already legally bound to perform is not consideration.

A junior associate at a law firm, prone to compulsive gambling and riddled with law school debt, threatened to file a civil suit against his law firm for discrimination after being passed over for a promotion. In actuality, the associate knew that he was never up for the promotion due to poor work performance, but the firm did not want the case to go to trial. The firm offered to settle with the associate for $10,000 in exchange for a promise not to sue. The associate agreed. However, the firm's senior partner changed her mind and the firm later repudiated the settlement agreement. The associate then filed suit against the firm for breach of contract. At trial, it was established that the firm did not discriminate against the associate, that the associate knew he was not up for a promotion, that the recipient of the promotion and several other associates had demonstrably superior job performance to the junior associate, and that the associate's suit was motivated by his need to pay his debts. Is the associate entitled to recover under the settlement agreement? A. No, because the consideration given for the settlement agreement was inadequate. B. No, because there was no bargained-for exchange to support the settlement agreement. C. Yes, because public policy favors the settlement of litigation. D. Yes, because there was a legal detriment bargained for in exchange for a promise.

B. No, because there was no bargained-for exchange to support the settlement agreement. Why? *A settlement agreement is a contract, so it can only be enforced if it is supported by consideration. For consideration to exist, (1) there must be a bargained-for exchange of promises or performance and (2) the promised or provided performance must have legal value. A party's promise not to bring suit to enforce a claim or assert a defense has legal value—and constitutes consideration for a settlement agreement—only if the claim or defense is valid or subject to a good-faith dispute or the party honestly believes that the claim or defense may be valid.

A motorcycle enthusiast purchased a custom-made motorcycle from a boutique motorcycle shop. The enthusiast paid $5,000 for the motorcycle, which was to be delivered to him in one month. One week after he purchased the motorcycle, the enthusiast decided that he no longer wanted it. He called up his best friend and told him that he would give him the motorcycle when it was finished. The enthusiast also instructed the shop to deliver the motorcycle to his friend. Three weeks later, the shop delivered the motorcycle to the enthusiast. The enthusiast accepted the motorcycle without protest and called to inform the friend that he had decided to keep the motorcycle. When the friend heard this, he sued the shop for its failure to deliver the motorcycle to him. Will the friend likely succeed? A. No, because an assignee cannot force an obligor to comply with an assignment by the obligee-assignor. B. No, because there was no consideration to support the enthusiast's assignment of the motorcycle to his friend. C. Yes, because the enthusiast made a valid assignment of the motorcycle to the friend. D. Yes, because the friend detrimentally relied upon the enthusiast's assignment.

B. No, because there was no consideration to support the enthusiast's assignment of the motorcycle to his friend. Why? *An assignment of rights made without consideration is a gratuitous assignment and can be revoked by the assignor unless (1) the obligor has already performed, (2) a document symbolizing the assigned right has been delivered, (3) a written assignment signed by the assignor has been delivered, or (4) the assignee has detrimentally relied on it.

Regarding modification of a contract for the sale of goods, which of the following statements is FALSE? A. The pre-existing duty rule does not apply. B. Past consideration constitutes consideration. C. A contract modification requires only good faith. D. A contract modification is binding without new consideration.

B. Past consideration constitutes consideration.

The owner of a retail clothing store regularly displayed for-sale works by local artists on a wall in the store. An art collector who came into the store inquired about purchasing a particular work for display at his home. The two agreed upon a price, but the collector was not ready to commit to purchasing it immediately. Confident that the collector would purchase the work, the owner promised in a signed writing to sell the work to the collector at the agreed-upon price at any time before the end of the month. On the last day of the month, the collector sent the owner a check for the agreed-upon price, which the owner received on the following day. If the owner returns the collector's check and refuses to sell the artwork to the collector, which of the following best supports the owner's position that a contract had not been formed? A. The collector could not accept the owner's offer by mailing a check. B. The collector's acceptance of the owner's offer was not timely. C. The firm-offer rule is not applicable because the collector was not a merchant with respect to the artwork. D. The firm-offer rule is not applicable because the owner was not a merchant with respect to the artwork.

B. The collector's acceptance of the owner's offer was not timely. Why? *Acceptance of a firm offer, option, or other irrevocable offer is effective only when it is received by the offeror. The mailbox rule does not apply.

A construction company contracted with a manufacturer to purchase 100 identical prefabricated windows to use while constructing houses in a gated community. The windows were to be delivered in shipments of 25 windows each on April 1, May 15, July 1, and August 15. The written contract, signed by both parties, was silent as to when payment for each shipment would be due. The manufacturer made the first two shipments in conformity with the contract requirements, and the construction company paid one-fourth of the full contract price upon each delivery. However, on June 1, the manufacturer demanded that the construction company pay the entire remainder of the contract price before the manufacturer made any further shipments. Which of the following statements is true? A. The construction company has no duty under the contract to make any payments until the final delivery is made. B. The construction company must pay the manufacturer one-fourth of the contract price upon delivery of each conforming shipment of windows. C. The construction company's failure to pay the requested sum will amount to a repudiation of the contract. D. The manufacturer waived his right to demand immediate payment of the full contract price when he accepted the first payment of one-fourth of the contract price on April 1.

B. The construction company must pay the manufacturer one-fourth of the contract price upon delivery of each conforming shipment of windows. Why? *Under the UCC, an installment contract is defined as a contract in which the goods are to be delivered in multiple shipments, and each shipment is to be separately accepted by the buyer. Payment by the buyer is due upon each delivery unless the price cannot be apportioned.

A jeweler and a goldsmith signed a written agreement that provided as follows: "For $3,000, the goldsmith shall sell to the jeweler a size six gold ring setting that the jeweler shall select from only the goldsmith's white gold ring designs." The agreement did not address any other specific terms with regard to the business arrangement between the jeweler and the goldsmith. When the jeweler arrived to select a ring, he refused to select one of the goldsmith's white gold ring designs. The jeweler claimed that the goldsmith, immediately prior to the execution of the written agreement, had orally agreed to broaden the jeweler's choices to also include rose gold ring designs. The jeweler also claimed that the goldsmith had, at the same time, orally agreed to include a set of earring settings, valued at $1,000, as an incentive for the jeweler's continued business. The goldsmith refused to sell to the jeweler any of his rose gold ring designs or include the earring settings. If the jeweler sues the goldsmith for damages, how should the court handle the evidence of the alleged oral agreements? A. The court should admit the evidence as to both the promise to include the earring settings and the option to choose a rose gold ring design. B. The court should admit the evidence as to the promise to include the earring settings but not the option to choose a rose gold ring design. C. The court should admit the evidence as to the option to choose a rose gold ring design but not the promise to include the earring settings. D. The court should exclude the evidence as to both the option to choose a rose gold ring design and the promise to include the earring settings.

B. The court should admit the evidence as to the promise to include the earring settings but not the option to choose a rose gold ring design. Why? The UCC presumes that a written contract is partially integrated. As a result, evidence that supplements the written contract is admissible—but evidence that contradicts the writing is inadmissible—under the parol evidence rule. *This means that the goldsmith's prior oral statement to include a set of earring settings merely supplements the writing and is not barred by the parol evidence rule.

A dancer signed a contract with a traveling circus to travel and perform as an aerialist for six months. The contract provided that the dancer would be paid $500 per week and would be guaranteed employment for the full six months, with an option to renew the contract for the next traveling season. Excited for the opportunity to perform for a traveling circus, the dancer turned down an invitation to dance with a theatre group for the same time period as the circus contract. After two weeks of traveling and dancing for the circus, the dancer sprained her ankle and was briefly hospitalized for one week. The circus was forced to hire another aerialist. After an additional week, the dancer's doctor gave her approval to return to work, but the circus refused to honor the remainder of the contract. The dancer brought an action against the circus for breach of contract. If the dancer wants to recover the highest possible amount of damages, which of the following is the dancer's best legal theory? A. The dancer detrimentally relied on the contract by declining the other dancing job. B. The dancer's failure to perform for two weeks was not a material breach of the contract. C. The dancer's performance of the terms of the contract was impracticable given her injury. D. The dancing contract with the circus is legally severable into weekly units.

B. The dancer's failure to perform for two weeks was not a material breach of the contract. Why? *A party who substantially performs contractual obligations can generally recover the contract price minus any cost that the nonbreaching party incurred to receive full performance. In contrast, a party who commits a material breach can recover only for any benefit conferred on the nonbreaching party minus damages for the breach.

FILL IN THE BLANKS. For the parol evidence rule to apply, there must be a(n) __________________ agreement and a(n) ________________ written or oral statement about the agreement. A. Written, subsequent B. Written, earlier C. Oral or written, subsequent D. Oral or written, earlier

B. Written, earlier

A couple, who wanted to open a pet grooming and supply store, contracted with a developer to lease space in a small strip mall that the developer was constructing. The lease was to begin on July 1, but on June 20, the developer informed the couple that the mall would not be finished, nor would the space be available, until August 1. The developer indicated that the first month's rent would be waived but that, because the lease did not contain a liquidated damages clause, he was not responsible for any damages attributable to the delay. As a consequence of the delay, the couple incurred storage costs and additional advertising expenses of $3,000. They also estimated in good faith that they lost $10,000 in sales. Which of the following amounts is the couple entitled to recover from the developer for the delay? A. Nothing, because the damages suffered by the couple's new business are too speculative. B. Nothing, because the lease did not contain a liquidated damages clause. C. $3,000, the amount incurred as a consequence of the delay. D. $10,000, the good-faith estimate of lost sales.

C. $3,000, the amount incurred as a consequence of the delay. Why? *Incidental damages may be awarded as compensation for commercially reasonable expenses incurred because of the other party's breach. And consequential damages may be awarded to compensate for losses that do not flow directly and immediately from the other party's breach—so long as the losses are not too speculative.

To which of the following situations does the parol evidence rule most likely apply? A. A party seeks to introduce evidence about a subsequent agreement. B. A party seeks to introduce the other party's oral assertion made during the negotiations of the agreement as evidence of mistake. C. A party seeks to introduce evidence of a written statement made before the written contract was entered into that directly contradicts a provision in that contract. D. A party seeks to introduce evidence of a second, separate deal.

C. A party seeks to introduce evidence of a written statement made before the written contract was entered into that directly contradicts a provision in that contract.

When the builder breaches a building contract, which of the following is FALSE? A. Expectation damages generally take into account the cost to complete the building in accordance with the contract. B. Expectation damages may be limited to the diminution in market value when economic waste would result. C. Damages based on the diminution in market value of the building may be awarded to the nonbreaching party whenever the breaching party acted willfully. D. Damages based on the diminution in market value may be awarded if the cost-to-complete damages will dramatically overcompensate the nonbreaching party.

C. Damages based on the diminution in market value of the building may be awarded to the nonbreaching party whenever the breaching party acted willfully.

An honest dispute developed between a condominium owner and a plumber over whether plumbing installed in the kitchen and bathrooms of the condominium satisfied contractual specifications. If the plumbing met those specifications, the condominium owner would owe the plumber $15,000 under the terms of the contract. The condominium owner offered to pay the plumber $10,000 in satisfaction of the owner's contractual obligations if the plumber replaced the plumbing in the kitchen with another grade of pipe. The plumber accepted the condominium owner's offer. After the plumber replaced the kitchen plumbing, the condominium owner refused to pay the plumber. In a breach-of-contract action brought by the plumber, the fact finder determined that the plumbing originally installed by the plumber did satisfy the contract specifications. The fact finder also determined that the plumber and the condominium owner entered into a substitute agreement under which the owner failed to deliver the required performance. What is the maximum amount that the plumber can recover in damages from the condominium owner? A. $25,000. B. $15,000. C. $10,000. D. Nothing.

C. $10,000. Why? A substitute contract is a second agreement that immediately discharges the original contract such that the remedy for breach is limited to the terms of the second contract.

A licensing agreement provided that a manufacturer could use an inventor's patent in manufacturing its products for 10 years. Immediately thereafter, the inventor assigned his rights to receive payments pursuant to the licensing agreement to a corporation. The inventor did not receive compensation for this assignment. The inventor, upon his death five years later, devised his stock in the corporation to his daughter and all of his remaining property to his son. To whom should the manufacturer make its payments under the licensing agreement? A. The corporation. B. The inventor's daughter. C. The inventor's son. D. No one, because the manufacturer's obligation to make payments under the licensing agreement terminated upon the death of the inventor.

C. The inventor's son. Why? *A gratuitous assignment—i.e., an assignment that is not supported by consideration—is automatically revoked upon the death, incapacity, or bankruptcy of the assignor.

Owners of hot water heaters sued the manufacturer in a class action lawsuit for breach of warranty. Under the terms of the settlement agreement, the owners were entitled to free repair of their hot water heaters from an authorized service provider, with the manufacturer promising to pay the service provider for performing the repair. One such authorized service provider repaired over 2,500 hot water heaters. When the service provider submitted a bill for its repair services, the manufacturer refused to pay because the owners had failed to follow the protocol for obtaining repair as established in the class action agreement. The service provider sued the manufacturer for breach of its agreement with the hot water heater owners. Is the service provider entitled to recover from the manufacturer? A. Yes, because the service provider was a third-party beneficiary of the agreement between the manufacturer and the hot water heater owners. B. Yes, because the service provider's rights under the agreement vested when the service provider repaired the hot water heaters. C. No, because the owners failed to satisfy a condition precedent to the manufacturer's duty to pay for the repairs. D. No, because the owners were not obligated to the service provider at the time that the agreement between the owners and the manufacturer was entered into.

C. No, because the owners failed to satisfy a condition precedent to the manufacturer's duty to pay for the repairs. Why? *When a party is suing as a third-party beneficiary of a contract, the promisor can raise any defense against the third-party beneficiary that the promisor had against the original promisee. Here, because the owners failed to satisfy a condition precedent to the obligation of the manufacturer to pay for the repair of their hot water heaters, the manufacturer's obligation did not arise. Consequently, the service provider was not entitled to payment from the manufacturer for its repairs of the hot water heaters.

A furniture manufacturer contracted with an office manager for the sale of a large couch for the office manager's lobby. The contract included the language "F.O.B. manufacturer's warehouse," and required delivery of the couch by August 5. On August 2, the manufacturer placed one of its large couches onboard a commercial carrier truck at its warehouse, properly contracted for its shipment to the office manager by August 5, and informed the office manager that the couch was on its way. The next day, while in transit, the truck carrying the couch was in a highway accident that completely destroyed the couch. When the office manager did not receive the couch on August 5, she called the manufacturer to demand another couch. The manufacturer refused, and the office manager brought an action against the manufacturer for breach of contract.Is the office manager likely to succeed in her action against the manufacturer? A. Yes, because the risk of loss for non-identified goods destroyed without the fault of either party remains with the seller. B. Yes, because the couch was never properly delivered to the office manager. C. No, because the risk of loss had already passed to the office manager when the couch was destroyed. D. No, because the manufacturer was not at fault for the destruction of the couch.

C. No, because the risk of loss had already passed to the office manager when the couch was destroyed. Why? *If the contract does not specify a place of delivery, it is a shipment contract (often identified by the words "F.O.B. (free on board) seller's place of business"), and the risk of loss passes to the buyer when the seller gives possession of the goods to the carrier, makes a proper contract for their shipment, and gives the buyer notice that the goods have been shipped. Therefore, in this situation, the risk of loss had already passed to the office manager at the time the couch was destroyed, and the manufacturer has fully performed on the contract.

A homeowner entered into oral contracts with both a painter and a landscaper to perform services at his home. The landscaper was the first to begin the services, and shortly after he began to work, he realized that the projected cost of the project would increase dramatically. After the homeowner learned how high the cost of the landscaping services was going to be, he called the painter to tell her that he could not go through with their contract at that time. The painter stated that she had already purchased a standard set of paintbrushes to paint his home, as well as glass necessary to create a custom mosaic on a back corner of the house, according to the homeowner's specifications. She had also paid for a temporary city permit to park her utility van on the residential street where the homeowner lived. In a suit by the painter against the homeowner, which of the following is the painter LEAST likely to recover? A. The contract price minus the market cost of performance. B. The cost of the glass for the mosaic. C. The cost of the paintbrushes. D. The cost of the parking permit.

C. The cost of the paintbrushes.

A homeowner entered into a written contract with a contractor to construct an elaborate tree house among the large trees located in the homeowner's backyard. After commencing construction of the tree house, the contractor discovered that one of the trees intended to be used as support for the tree house had a relatively common fungal infection in its core that would cause the strength of the tree's branches to falter if left untreated. Neither the homeowner nor the contractor had knowledge of the fungal infection when they entered into the contract, but the contractor knew that such infections were common in the area and did not request an inspection of the trees before entering the contract. The contractor also knew that treatment was available at a high cost, but that even after treatment, he would need to create additional heavy-load-bearing supports for the tree at a substantial cost. When the contractor informed the homeowner that he would not perform under the contract unless the homeowner provided at least 75% of the additional costs needed to make the structure safe, the homeowner refused to pay the additional amount. The homeowner then sued the contractor for breach of contract. What is the likely result? A. The contractor wins, because his performance was discharged due to impracticability. B. The contractor wins, because neither party was aware of the fungal infection. C. The homeowner wins, because the contractor assumed the risk of the fungal infection. D. The homeowner wins, because the fungal infection did not render performance impossible.

C. The homeowner wins, because the contractor assumed the risk of the fungal infection.

A homeowner met with a number of general contractors regarding significant renovations on his home. After a few successful meetings with one particular contractor, the homeowner received a letter from the contractor stating that he would perform the renovation work for $10,000. The homeowner responded with a letter stating that he would "only pay $8,500 and no more." Upon receiving the homeowner's letter, the contractor immediately bought the materials necessary to complete the renovation and began renovating the homeowner's home. One week after beginning work on the homeowner's home, the contractor realized that the renovation would cost $1,600 more than he had anticipated due to the homeowner's very particular aesthetic requirements. After being informed of the higher cost, the homeowner refused to pay the contractor any amount over $8,500. The contractor promptly discontinued working on the renovation. The homeowner subsequently brought a breach-of-contract action against the contractor, seeking damages. Which party will likely prevail? A. The contractor, because the homeowner's offer could only be accepted by a return promise. B. The contractor, because the unexpected costs discharged his obligation to complete performance. C. The homeowner, because his offer was effectively accepted by commencement of performance. D. The homeowner, because the unilateral contract between the parties is enforceable.

C. The homeowner, because his offer was effectively accepted by commencement of performance. Why? *An offer is presumed to be bilateral and can be accepted with a return promise or by starting performance. In contrast, a unilateral offer can only be accepted by completing performance.

A fashion designer wanted to start selling her sneakers, so she contacted a clothing manufacturer to make the sneakers in accordance with her designs. After negotiating the price and other terms, the two parties entered into a written contract on January 1. Under the contract, the manufacturer promised to make the sneakers on the condition that the designer send the design specifications to the manufacturer by February 15. The manufacturer made clear to the designer that the first batch of sneakers would take at least two months to manufacture after receipt of the design specifications. The manufacturer also promised to deliver the first batch of sneakers to the designer by May 15. On February 27, the designer sent the design specifications to the manufacturer without having sent the manufacturer any prior notification that there would be a delay. Despite the delay, the manufacturer proceeded to manufacture the first batch of sneakers according to the specifications, and they were completed by May 1. However, due to a paperwork error by the manufacturer, it failed to deliver the sneakers to the designer until June 1. Which most accurately describes the legal obligations of the two parties under the contract on June 1? A. The designer's failure to submit specifications by February 15 excused the manufacturer's duty to deliver the sneakers by May 15. B. The designer's failure to submit specifications by February 15 waived the condition that the manufacturer would deliver the sneakers by May 15. C. The manufacturer was in breach of the contract for failing to deliver the sneakers by May 15. D. There was an implied condition that the designer give the manufacturer notice of any delay in submitting the specifications.

C. The manufacturer was in breach of the contract for failing to deliver the sneakers by May 15. Why? *The nonoccurrence of a condition may be excused if the party who benefits from the condition waives it by words or conduct. Once the condition is excused, then the waiving party cannot raise it as a defense.

A dairy ordered a tank trailer from a commercial retailer. The contract did not identify a specific tank trailer, but did specify various features that the tank trailer should have. The contract called for the retailer to deliver the tank trailer to the dairy. Subsequently, the retailer acquired a tank trailer from a manufacturer that it believed met the requirements of the contract. Several days later, the retailer, in accordance with the contract, had the dairy's emblem painted on each side of the tank trailer, and then delivered it to the dairy. The tank trailer did not, in fact, conform to the contract. At which time did the dairy acquire an insurable interest in the tank trailer? A. When the contract was entered into. B. When the retailer acquired the tank trailer from the manufacturer. C. When the retailer had the dairy's emblem painted on the tank trailer. D. When the retailer delivered the tank trailer.

C. When the retailer had the dairy's emblem painted on the tank trailer. Why? *A buyer acquires an insurable interest in goods upon the identification of the goods. Where the contract is for future goods (i.e., goods that are not both existing and identified), the buyer does not acquire an insurable interest until the seller designates goods as those to which the contract refers, unless the parties have explicitly agreed otherwise. Since the dairy did not select a tank trailer from the retailer's inventory or otherwise select a specific trailer, the contract was for the sale of future goods. By painting the dairy's emblem on the tank trailer, the seller effectively designated that tank trailer as the dairy's. Consequently, at that time, the dairy acquired an insurable interest in it. Although the trailer's failure to conform to the contract does give the dairy rights against the retailer, the failure does not affect the identification of the trailer.

On April 1, a buyer and a seller executed a written contract for the sale of an antique car for $40,000, delivery on May 1. The contract contains a clause indicating that it is a total integration of the parties' agreement. As they each signed the contract, the buyer orally reminded the seller that the buyer's duty to purchase the car was conditioned on his ability to get approval for a loan by April 20 to fund the purchase. The seller orally agreed, though the condition was not noted in the written contract. When the seller contacted the buyer to execute the sale on May 1, he discovered that the buyer had attempted but failed to get a loan and could not afford to purchase the car. The buyer refused to honor the contract. If the seller sues the buyer for breach of contract, will the court likely admit the evidence of the oral condition regarding the buyer's approval for a loan? A. No, because the oral agreement contradicts the terms of the written contract. B. No, because the written contract is a complete integration of the agreement between the parties. C. Yes, as proof of a condition precedent to the buyer's obligation under the contract. D. Yes, because the oral agreement was a distinct and separate contract.

C. Yes, as proof of a condition precedent to the buyer's obligation under the contract. Why? *Evidence used to establish a condition precedent that must occur before a contract becomes effective is admissible under an exception to the parol evidence rule.

A wedding planner contracted with a local bakery to make cupcakes for an upcoming wedding reception. The bakery was very experienced in making cupcakes and had a great reputation in the community. Although there were other comparably skilled cupcake makers in the area, the wedding planner eventually chose the bakery due to the price it quoted her for the cupcakes. A few months before the wedding reception, the bakery's head baker unexpectedly had to take a leave of absence to deal with a medical issue. The bakery subsequently assigned the contract to a pastry chef in the same community. The pastry chef also had an excellent reputation in the community and was at least equally as skilled at making cupcakes as the bakery. The bakery told the wedding planner about this assignment, and the wedding planner did not object. When the pastry chef fully performed on the contract and delivered the cupcakes, which conformed to the contract requirements, the wedding planner refused to accept or pay for the cupcakes. On these facts, has the wedding planner breached the contract? A. No, because the bakery breached the contract first by assigning the contract to another party. B. No, because the wedding planner only had a duty to accept performance by the bakery. C. Yes, because the assignment of the contract was permitted and the pastry chef properly and fully performed. D. Yes, because the bakery told the wedding planner about the assignment of the contract and the wedding planner did not object.

C. Yes, because the assignment of the contract was permitted and the pastry chef properly and fully performed. Why? *Delegation of contractual duties is not permitted when (1) the contract prohibits delegation or (2) the other party to the contract has a substantial interest in having the delegating party perform. Otherwise, delegation is permitted, and the other party to the contract must accept performance by the delegatee.

In June, a local chef learned of a new business that opened in the area. Hoping to attract the business as a new client, the chef sent the business an offer consisting of a catalog of menus available through his catering service and a form letter that he sent to all new businesses in the area. The letter was signed by the chef and included the following language: "Welcome! I specialize in creating delicious meals with local and organic ingredients, and I would be honored to be your catering source for all your business, promotional, and personal needs! To welcome you to the community, I would like to offer you a 25% discount off my catalog prices on any three-course meal order, for up to 100 people, submitted this calendar year. I hope to hear from you soon, and I look forward to doing business with you!" No communication occurred between the parties until the end of November, when the business faxed an order form to the chef requesting a catered meal for 60 people at a promotional event for a 25% discount. The chef refused to provide the business with the catered meals at a 25% discount. As a result, the business sued the chef for breach of contract. The court found that both parties are merchants with respect to this transaction. Is the business likely to succeed in its action? A. No, because the business's power of acceptance terminated after a reasonable period of time. B. No, because the form letter was only an invitation to deal. C. Yes, because the chef had not revoked the offer before the end of the calendar year. D. Yes, because the signed promotional letter created a firm offer.

C. Yes, because the chef had not revoked the offer before the end of the calendar year. Why? *An offer terminates by lapse if it is not accepted by a specified date or, if no date is specified, after a reasonable time. And a revocable offer terminates by revocation if it is revoked by the offeror prior to acceptance.

A widow offered to sell her small business, together with all of the business's assets, to a nonprofit organization. The organization accepted, and on June 1, it signed and executed a contract providing for the sale of the business for $25,000 at the end of the month. When the organization's agent signed the contract, she orally informed the widow that the organization's duty to purchase the business was conditioned on obtaining approval from a local zoning board to convert the business's primary office into an affordable-healthcare clinic. A week later, the woman received another offer to purchase her business for $35,000. At the end of the month, seeking to accept the other offer, the widow refused to honor the contract with the organization because it had neglected to request the necessary approval from the zoning board. The organization sued the widow for breach of contract. The organization presented clear evidence that it had the necessary funds to perform on the contract at the end of the month, and that the zoning board would have routinely approved the organization's plans for the office. Is the organization likely to prevail in its action against the widow? A. No, because the express condition of the zoning board's approval had not occurred by the end of the month. B. No, because the organization's failure to seek approval from the zoning board was a repudiation of the contract. C. Yes, because the condition of approval by the zoning board has been waived by the organization. D. Yes, because the condition of approval by the zoning board was not included in the written contract.

C. Yes, because the condition of approval by the zoning board has been waived by the organization. Why? *A party to a contract whose duty to perform is subject to a condition can waive the condition by words or conduct.

A charity, seeking to raise funds, held a legally permitted raffle in which the prize was a new automobile. A week before the raffle, the organizer of the raffle contacted a friend who had purchased a raffle ticket. The organizer promised to ensure that the friend would win the raffle if the friend gave the organizer $1,000. The friend agreed and gave the organizer $1,000. On the day before the raffle, the friend began to feel guilty. He went to the organizer, renounced the scheme, and demanded his $1,000 back. The organizer refused. The next day at the raffle, the automobile was awarded to someone else. The applicable jurisdiction makes it a crime to fraudulently conduct a contest, lottery, or prize drawing. If the friend sues the organizer for the $1,000, will the friend be likely to prevail? A. No, because the agreement between the friend and the organizer was illegal. B. No, because the friend failed to take any action to prevent the raffle from being held. C. Yes, because the friend is entitled to a return of the $1,000 paid to the organizer. D. Yes, because there was a valid contract between the organizer and the friend.

C. Yes, because the friend is entitled to a return of the $1,000 paid to the organizer. Why? *A party to an illegal contract may recover restitution damages if that party conferred a benefit on the other party and (1) was justifiably ignorant of the facts that made the contract illegal, (2) was less culpable than the other party, or (3) withdrew before the contract's illegal purpose was achieved and did not engage in serious misconduct.

A sister convinced her brother that they should open a small coffee shop. Their friend, a guitarist, suggested bringing his band to play live music in order to attract customers. He did not request any payment, saying that the publicity would be good for the band. The siblings agreed, and the band started playing at the coffee shop weekly. The coffee shop became a success, in no small part due to the band's performances. When a businessperson offered to buy the coffee shop from the siblings, they orally agreed to each pay the guitarist $10,000 out of their share of the sale proceeds for his help in making the shop popular. The sister told the guitarist about their agreement. He was so delighted with it that he made a down payment on a new car. By the time the sale of the coffee shop was finalized, the brother had encountered financial difficulties. After the sale, the siblings signed a written contract stating that the sister would pay the guitarist $10,000 and her brother would pay him $5,000. If, after the sale, the brother pays the guitarist only $5,000, will the guitarist have a valid basis for an action against the brother for another $5,000? A. No, because the guitarist was bound by the written modification of the contract made by the siblings. B. No, because the guitarist was only a donee beneficiary of the oral contract between the siblings. C. Yes, because the guitarist's reliance on the promised payment prevented the siblings from changing the obligations of their oral contract. D. Yes, because the oral promise to pay $10,000 to the guitarist was made binding by the guitarist's valuable and uncompensated contributions to the business.

C. Yes, because the guitarist's reliance on the promised payment prevented the siblings from changing the obligations of their oral contract.

On May 1, a clothing manufacturer sent a written offer to a retailer for the sale of 1,000 pairs of designer jeans at a price of $50 per pair, including delivery and transportation costs. The proposed delivery date was June 15. On May 5, the retailer mailed a letter accepting the manufacturer's offer. On May 15, a natural disaster occurred, causing fuel prices to significantly increase. As a result, the manufacturer sent a letter to the retailer requesting an additional $500 to cover the increased delivery and transportation costs for the shipment of the jeans. The retailer considered seeking out a new supplier. However, the retailer hoped to continue doing business with this manufacturer in the future, so she decided against it. The retailer returned a signed letter promising to pay the additional $500. On June 15, the manufacturer delivered 1,000 pairs of jeans to the retailer. The retailer paid the manufacturer $50,000 but refused to pay the additional $500. Can the manufacturer enforce the retailer's promise to pay an additional $500? A. No, because no consideration supported the promise to pay the additional $500. B. No, because the attempted modification was unconscionable. C. Yes, because the manufacturer made its request for the additional $500 in good faith. D. Yes, because the manufacturer relied on the promise when it delivered the jeans to the retailer.

C. Yes, because the manufacturer made its request for the additional $500 in good faith. Why? *Under the UCC, no consideration is needed to modify a contract. All that is required is good faith—i.e., honesty in fact and fair dealing per reasonable commercial standards.

On October 1, a retail sporting equipment store telephoned a shoe manufacturer and offered to buy a minimum of 50 and a maximum of 100 pairs of running shoes at $40 a pair to be delivered in 60 days. The manufacturer orally accepted the offer and then immediately faxed a signed letter to the store. The letter contained the following language: "This letter confirms our agreement by telephone on October 1 to sell you 50 pairs of running shoes for 60-day delivery." Sixty days later, the manufacturer delivered 100 pairs of conforming running shoes to the store. However, the store rejected all 100 pairs of shoes because it had found the same shoes from another seller for $30 a pair. Due to lack of demand, the manufacturer cannot resell the 100 shoes without suffering an economic loss. Can the manufacturer enforce a contract against the store? A. No, because the manufacturer did not state the agreed-upon price term in the faxed letter. B. No, because the store did not sign the faxed letter. C. Yes, for the sale of 50 pairs of shoes, because the manufacturer's faxed letter stated that quantity term. D. Yes, for the sale of 100 pairs of shoes, because the store wrongfully rejected that quantity of properly delivered and conforming shoes.

C. Yes, for the sale of 50 pairs of shoes, because the manufacturer's faxed letter stated that quantity term. Why? *Under the UCC, when both parties are merchants, a new or additional term in the acceptance become part of the contract unless (1) the offer limits acceptance to the offer's terms, (2) the new term materially alters the contract, or (3) the offeror objects to the new term within a reasonable time.

A tenant rented a small cabin from a landlord. The lease provided that the tenant was permitted to make structural improvements to the cabin but that the tenant must pay for such improvements. Relying on this clause in the contract, the tenant contacted a contractor to install a loft in the cabin for $10,000. The tenant and the contractor agreed in a writing signed by both parties that payment would be due 30 days after the loft was completed. The contractor knew that the tenant was renting the cabin and sent the landlord a letter informing him of the impending construction on his property. The landlord received the letter and did not reply. The contractor completed the loft, which increased the market value of the cabin by $6,000. Ten days later and three months before the end of her lease, the tenant vacated the cabin and disappeared. Thirty days after the loft was completed, the contractor's bill remained unpaid. If the contractor has no remedy quasi in rem under the jurisdiction's mechanic's lien statute, which of the following will give the contractor the best chance of recovery in personam against the landlord? A. An action as a third-party beneficiary to the lease. B. An action based on an implied-in-fact contract. C. An action based on promissory estoppel. D. An action in quasi-contract for the benefit conferred on the landlord.

D. An action in quasi-contract for the benefit conferred on the landlord. Why? *A plaintiff can recover under a quasi-contract theory—despite having no contractual relationship with the defendant—if the plaintiff conferred a non-gratuitous benefit on the defendant that resulted in unjust enrichment.

On January 3, a fruit grower entered into a written agreement to sell all of the cherries and peaches to be grown in her orchard that year to a farmers' cooperative association. The agreement specified $0.50 per pound for cherries, to be delivered by April 1, and $0.70 per pound for the peaches, to be delivered by May 15, and provided that the grower was to be paid only once at the end of the season for harvesting both fruits. The cherries had been picked and delivered according to schedule, but on May 15, the grower told the association that she would not pick and deliver the peaches, which were not yet ripe. The grower pointed out that she and the association had mistakenly underestimated the current market price of peaches. She also informed the association that her son, whom she was counting on to supervise the pickers in exchange for a percentage of the profits, had unexpectedly accepted a job offer to begin working elsewhere, making it unlikely that she could find a replacement who would agree to a similar delayed compensation arrangement. She contended that the combination of these two events had effectively made it impossible for her to earn a profit from the harvest of her peaches. The fruit grower contends that she is entitled to payment for the cherries that she picked and delivered to the association. Which of the following constitutes the best grounds to the support the grower's demand for payment? A. Mutual mistake B. Impracticability C. Frustration of purpose D. Divisibility of the contract

D. Divisibility of the contract Why? *A divisible contract is one in which the obligations imposed on the contracting parties can be separated into corresponding pairs of part performances such that each pair constitutes agreed equivalents. Here, although the contract provides for only a single payment at the end of the season for harvesting both the cherries and the peaches, the amount that the association is obligated to pay the grower for the cherries that she picked and delivered is computed separately from the amount for the peaches, and each fruit type was to be delivered separately. Consequently, although the written agreement requires the grower to deliver the peaches and the cherries from her orchard prior to being paid, because the amount paid for the cherries is not dependent on the amount paid for the peaches, the grower's obligation to deliver the cherries and the association's obligation to pay for them constitute a separate corresponding pair of part performances. Because the grower has picked and delivered the cherries, she is entitled to payment for them, even though she likely has breached her obligation with regard to the peaches. Answer choice B is incorrect. Although the departure of the grower's son during harvest season may make it difficult to find a replacement who will assume his role as supervisor of the pickers, the grower is merely contending that it will result in a compensation arrangement with the replacement that is not as favorable as the one she had with her son. Generally, non-extraordinary increases in the cost of performance do not rise to the level of impracticability that would justify a discharge of the grower's obligation to pick the peaches in her orchard and deliver them to the association. Moreover, the risk of the grower's son finding a job elsewhere, while perhaps unforeseeable, is most likely a risk that the grower will be deemed to have assumed.

In applying the parol evidence rule to a sale of goods, which of the following is TRUE? A. There is a presumption that a written contract is a complete integration of the parties' agreement. B. The parol evidence rule does not apply to a sale of goods. C. The parol evidence rule applies only to an oral statement made before the written contract was entered into. D. Evidence of an additional promise made before the written contract was entered into that does not contradict the contract may be considered unless the parties would certainly have included the term in the contract.

D. Evidence of an additional promise made before the written contract was entered into that does not contradict the contract may be considered unless the parties would certainly have included the term in the contract.

An independent trucker and a manufacturer entered a written contract for the delivery of a farming implement from the manufacturer to a farmer. Under the terms of the contract, the trucker promised "to deliver a farming implement from the manufacturer to the farmer," and in exchange, the manufacturer promised "to pay the trucker if the trucker delivers the implement directly to the farmer after picking it up." The trucker picked up the implement but, instead of driving directly to the farmer, drove 100 miles out of his way to pick up another item from a third party before delivering the implement to the farmer. The manufacturer, unaware that the trucker had failed to deliver the implement directly to the farmer, refused to pay the trucker. Who has breached this contract? A. Both the trucker and the manufacturer. B. The trucker only. C. The manufacturer only. D. Neither the trucker nor the manufacturer.

D. Neither the trucker nor the manufacturer. Why? *If contracting parties expressly agree to a condition precedent—an uncertain future event that must occur before a party's obligation to perform arises—then performance is not due until the condition is fully satisfied.

A mining company contracted with a railroad to transport 10,000 tons of coal from the company's mines to a power company at a cost of $100,000. The railroad told the mining company that the coal would arrive at the power company on June 1, but the contract contained a clause that the railroad would not be liable for any losses suffered by the mining company as a result of a late shipment. The railroad was aware that the mining company had contracted with the power company to deliver the coal on June 1, and pursuant to standard industry custom, the price to be paid by the power company decreased by $1 per ton for each day that the coal was late. The shipment of coal did not reach the power company until June 11, and the railroad had no justification for the 10-day delay. Because of the delay, the mining company lost $100,000 in revenue from the sale. The mining company filed suit against the railroad for breach of contract, claiming $100,000 in damages. Is the mining company likely to succeed in its claim? A. Yes, because the damages that the mining company would suffer from the railroad's delay were known to the railroad prior to shipment of the coal. B. Yes, because consequential damages cannot be excluded by a merchant. C. No, because the claimed damages are disproportionate to the original contract price between the railroad and the mining company. D. No, because the contract between the mining company and the railroad protected the railroad from liability for losses suffered by the mining company due to a late shipment.

D. No, because the contract between the mining company and the railroad protected the railroad from liability for losses suffered by the mining company due to a late shipment.

A company leased office space in a downtown building and subsequently entered into a written contract with a supplier to purchase furniture for the office. A dispute later arose over the tables and desks delivered by the supplier. The contract called for "cherry tables and desks" of designated designs. The company contended that the word "cherry" indicated the type of wood from which the tables and desks were made. The supplier, having delivered tables and desks made of a less expensive wood and finished with a cherry veneer, asserted that the use of the word "cherry" referred to the appearance of these items and did not require that the furniture be made solely of cherry wood. In the litigation of this dispute, the company sought to introduce a statement made by the supplier during negotiations that the tables and desks were of "solid-wood construction." In determining whether the parties intended the contract to be their final agreement, which of the following best reflects the rule of interpretation that the court should apply? A. The court can find that the contract is integrated only if it contains a merger clause. B. The court is permitted to look only within the "four corners" of the document for evidence of intent. C. The court must presume that the written contract is fully integrated. D. The court should presume that the written contract is partially integrated.

D. The court should presume that the written contract is partially integrated. Why? *Under the UCC, a court should presume that a written contract for the sale of goods is only partially integrated. As a result, evidence of additional consistent terms is admissible unless the court concludes that the parties certainly would have included those terms in the writing.

Which of the following is NOT a factor in determining whether a parent should be allowed to relocate with the child? A. The child's preference B. The ability to preserve the relationship with the nonmoving parent C. The age and needs of the child D. The court's ability to enforce the custody order

D. The court's ability to enforce the custody order

A man was moving to another state and decided that he wanted to give away some of his belongings. The man knew that his brother had always expressed interest in the man's antique desk. The man called the brother and said, "I'm going to be moving in two weeks. I would like to give you the antique desk as a gift. I'll drop it off at your house on my way out of town." The brother told the man that he was very grateful for the gift and was looking forward to having the desk in his home office. The brother, in reasonable reliance on the man's promise, immediately disposed of his old desk and made room for the antique one. A couple of days later, an appraiser, who was a friend of the man, visited the man's house for dinner. While at his house, the appraiser saw the antique desk and informed the man that it was worth well over $20,000. The man decided to keep the desk and did not drop it off at the brother's house on his way out of town. The brother brought suit against the man to recover the antique desk. If the court finds in favor of the man on these facts, what is the most likely reason? A. A promise to make a gift in the future cannot be enforced. B. The brother did not rely to his detriment on the man's promise. C. The man's promise was not in writing. D. The man's refusal to give the antique desk did not cause injustice.

D. The man's refusal to give the antique desk did not cause injustice. Why? *Under the doctrine of promissory estoppel, a party's promise to make a gift is enforceable if (1) the promisor should reasonably expect the promisee to rely on the promise, (2) the promisee detrimentally relies on the promise, and (3) injustice can be avoided only by enforcement of the promise.

On behalf of an elementary school, the school's principal entered into a written contract to purchase shirts for the school's students for a total cost of $5,000. The name of the school was to be imprinted on the back of each shirt. After the seller had acquired the shirts but before they had been imprinted, the principal emailed the seller and requested, in good faith, that a picture of the school's mascot be imprinted on the front of the shirts at no additional cost. In a reply email, the seller agreed to the principal's request. When the shirts arrived at the school, only the school's name appeared on the back of each shirt; the school's mascot did not appear on the shirt. The principal rejected the shirts and refused to pay for them. The seller sued the school for breach of contract. Who will prevail? A. The seller, because the shirts were specially manufactured goods. B. The seller, because the seller did not receive consideration for the modification. C. The school, because the school was not a merchant. D. The school, because of the perfect tender rule.

D. The school, because of the perfect tender rule. Why? *In a sale-of-goods transaction, such as the purchase of shirts, the Uniform Commercial Code (UCC) requires that the goods tendered by the seller conform in all respects to the contract. Although the shirts shipped to the school conformed to the specifications of the original contract, this contract was modified by the subsequent emails between the principal and the seller.

On January 5, a buyer and a seller contracted for the delivery of 100 widgets if they could be delivered by February 20. The agreement was made in a writing signed by both parties and provided that the buyer would pay the contract price of $1,000 upon delivery. On February 3, the buyer and the seller orally agreed to postpone delivery until March 1. However, when the widgets arrived on March 1, the buyer refused to accept or pay for the widgets. If the seller sues the buyer for breach of contract, who is most likely to succeed in the action? A. The buyer, because any modification of the parties' contract must satisfy the statute of frauds. B. The buyer, because the agreement on February 3 was not supported by consideration. C. The seller, because the contract modification on February 3 was immediately binding on both parties. D. The seller, because the oral agreement on February 3 waived the February 20 delivery date.

D. The seller, because the oral agreement on February 3 waived the February 20 delivery date. Why? *Nonoccurrence of a condition may be excused if the party who would benefit from the condition waives it by words or conduct. And that waiver cannot be retracted if the other party has detrimentally relied on it.

A refrigeration-unit manufacturer contracted with a kitchen appliance store to sell and deliver 100 refrigeration units to the store at a price substantially lower than market value. The written and signed contract included the term "F.O.B. kitchen appliance store, on or before March 30." The shipping company that the manufacturer normally used to deliver its refrigeration units experienced an unforeseen strike at the end of March. As a result, the manufacturer personally delivered the units to the store on April 18. The store suffered no material harm due to the delay. The refrigeration appliance industry generally allows appliance manufacturers a 30-day leeway for any contractually specified time of delivery, unless such leeway is expressly prohibited by the contract. If the store brings suit against the manufacturer for breach of contract, which of the following facts provides the manufacturer with the strongest defense to the store's claim? A. The delay was caused by an unforeseeable strike. B. The manufacturer believed that due to the price at which it offered the refrigeration units, the store would accept a late delivery. C. The store suffered no material harm from the delay. D. There is evidence of a trade usage in the refrigeration appliance industry allowing a 30-day leeway for appliance deliveries.

D. There is evidence of a trade usage in the refrigeration appliance industry allowing a 30-day leeway for appliance deliveries. Why? *Under the UCC, a party may explain or supplement the terms of a written contract with evidence of trade usage—i.e., any practice or method of dealing in the particular business or industry that is observed with such regularity so as to justify an expectation that it will be observed in the instant case.

A groom left his bride at the altar on the day of their wedding. The bride could not bear to keep any painful reminders of the occasion, so she offered to sell her wedding dress to one of her bridesmaids for $5,000. The bride stated that the offer would remain open for 30 days. The bridesmaid said that she was interested but would have to think about it. A week later, the bridesmaid emailed the bride to ask if the price included the custom-made veil that the bride had worn. The bride did not respond to the bridesmaid's question. Within the 30-day period, the bridesmaid accepted the bride's initial offer of $5,000 for the wedding dress. In response, the bride stated that the bridesmaid could only buy the wedding dress for $6,000. Was a contract formed when the bridesmaid accepted the initial offer of $5,000? A. No, because the bride raised the price of the dress to $6,000. B. No, because the bridesmaid's question acted as a counteroffer and a rejection of the $5,000 offer price. C. Yes, because the bride was required to keep the initial offer open for the 30-day period. D. Yes, because the bridesmaid's question did not constitute a counteroffer.

D. Yes, because the bridesmaid's question did not constitute a counteroffer. Why? *An offer that specifies a date on which it will terminate will automatically terminate on the specified date. But unless consideration was paid to keep this option open, the offer can be terminated at an earlier date—e.g., if the offeree makes a counteroffer.

A homeowner called a septic cleaning company and made arrangements for the company to remove the waste from the septic tank on the homeowner's property. After completing the job, the company mailed the homeowner a bill for $500, the fair market value of the services rendered by the company. The bill indicated that payment was due in 60 days. Upon receiving the bill, the homeowner called the company and informed it that, since he had lost his job due to an accident, he would not be paying the company's bill. The following day, the company filed suit for breach of contract. Ten days later, the homeowner moved to dismiss the suit. The court granted the motion, dismissing the suit without prejudice. Is the court's dismissal proper? A. No, because the parties' dealings created an implied-in-fact contract. B. No, because the homeowner has repudiated the contract. C. Yes, because the company failed to demand assurances. D. Yes, because the company's complaint is premature.

D. Yes, because the company's complaint is premature.

A manufacturer of T-shirts contracted with a brand-new clothing store to sell the store 1,000 T-shirts per month for a period of two years. The clothing store's signature color for its clothing was an orange-tinted red color, called coquelicot, which is very difficult to replicate on a consistent basis. The final, written contract specified that any T-shirts that were not coquelicot could be returned, but it was silent with regard to the return of T-shirts for other reasons. One year into the contract, the store decided to switch to coquelicot-colored baseball caps instead of T-shirts. As a result, the store returned the most recent shipment of coquelicot-colored T-shirts to the manufacturer and demanded a refund. The manufacturer refused to grant the refund, and the store sued the manufacturer for damages. At trial, the manufacturer introduced the contract, which clearly stated that T-shirts that were not coquelicot could be returned. The store then attempted to introduce evidence that it had returned coquelicot-colored T-shirts to the manufacturer over the past year without objection and received a refund. Is this evidence admissible? A. No, because evidence regarding the return of the T-shirts violates the parol evidence rule. B. No, because the express term in the contract regarding the return of T-shirts takes precedence over the course of performance. C. Yes, because the evidence can reasonably establish the parties' course of dealing on this issue. D. Yes, because the evidence is relevant to show that the manufacturer had accepted the return of coquelicot-colored T-shirts in the past.

D. Yes, because the evidence is relevant to show that the manufacturer had accepted the return of coquelicot-colored T-shirts in the past. Why? *Under the UCC parol evidence rule, course of performance can be used to supplement or explain the terms of a final written agreement.

A general contractor learned that a company was accepting bids for a lucrative construction project involving a high-rise building. The general contractor contacted a number of subcontractors and informed them that he would be accepting bids for the electrical work on the project for the next week. After receiving a number of bids from subcontractors, the general contractor selected a bid from a young subcontractor, which was the lowest bid but still within a reasonable range of the other bids. The general contractor used that sub-bid in calculating his overall bid on the construction project. Soon after submitting his sub-bid to the general contractor and after the general contractor had submitted his overall bid to the company, the young subcontractor realized that he could have charged more for his services based on their market value. The company ended up choosing the general contractor's bid for the project, and later that same day, the young subcontractor told the general contractor that he was revoking his sub-bid for the electrical work. As a result, the general contractor had to use a different subcontractor to perform the work at a cost $3,000 higher than the young subcontractor's bid. In a suit to recover the $3,000 from the young subcontractor, is the general contractor likely to prevail? A. No, because the young subcontractor's sub-bid was an offer that could be freely revoked. B. No, because the young subcontractor's sub-bid was lower than the market value of similar services. C. Yes, because an enforceable contract was formed when the general contractor used the young subcontractor's sub-bid in his overall bid on the project. D. Yes, because the general contractor detrimentally relied on the young subcontractor's sub-bid.

D. Yes, because the general contractor detrimentally relied on the young subcontractor's sub-bid. Why? *An offer is binding as an irrevocable option contract if (1) the offeror should have reasonably expected to induce reliance on the offer, (2) the offeree reasonably relied on the offer, (3) reliance caused the offeree to suffer substantial detriment, and (4) injustice can be avoided only by enforcing the offer.

A nature magazine advertised a photography contest in its January issue, offering "$1,000 to any subscriber who sends us a photograph of the rare Florida Grasshopper Sparrow that we use for the cover of our May issue. Only submissions meeting our technical specifications and received by April 1 will be considered." The only subscriber to respond to the advertised contest sent the magazine a photograph of the sparrow that met the magazine's technical specifications. The photograph arrived on March 15. However, due to an ecological disaster that occurred in early April, the magazine decided to use a different picture on the cover of its May issue. The magazine used the subscriber's picture on the cover of its June issue and has refused to pay $1,000 to the subscriber on the ground that it was not used on the May cover. Is the subscriber likely to prevail in a breach-of-contract action against the nature magazine? A. No, because the subscriber's photograph was not used on the cover of the May issue. B. No, because the subscriber failed to adequately notify the magazine of his acceptance. C. Yes, because all of the express conditions of the offer have been satisfied. D. Yes, because the magazine prevented the publication of the photograph.

D. Yes, because the magazine prevented the publication of the photograph.

On November 1, the owner of a yacht posted a flyer at a local coffee shop reading, "Yacht for Sale: Make me an offer!" The flyer also included the owner's phone number. A buyer called the owner on November 3 to ask how much the owner wanted for the yacht. The owner said, "Well, I'd hate to part with it for less than $55,000, but if you can pay me $50,000 by November 20, I'd sell it to you. I'll hold onto the yacht for you until then." Elated, the buyer took steps to obtain a loan by November 20. On November 15, a second buyer called the owner and offered to buy the yacht for $60,000. The owner immediately accepted, and the second buyer picked up the yacht the next day. On November 20, having obtained a loan, the first buyer visited the owner with a check for $50,000. The first buyer then learned the owner had already sold the yacht. Can the first buyer bring a successful suit against the owner for breach of contract? A. No, because the owner's statement to the first buyer was only an invitation to deal. B. No, because the second buyer offered more money for the yacht than the first buyer agreed to pay. C. Yes, because the owner promised to keep the offer open for a specific period of time. D. Yes, because the owner's offer to the first buyer was still outstanding on November 20.

D. Yes, because the owner's offer to the first buyer was still outstanding on November 20. Why? *An offer can be revoked by the offeror (1) expressly, when the offeror communicates the revocation directly to the offeree or (2) constructively, when the offeree acquires reliable information that the offeror has taken definite action inconsistent with the offer.

A plaintiff must prove racial intent or purpose in order to establish a violation of the 14th Amend.'s Equal Protection Clause. What must a plaintiff show to establish racial discrimination under the Fair Housing Act?

The plaintiff needs to show a disparate racial impact.


Related study sets

ECON120-Homework Questions and Answers

View Set

NURS 401 PrepU Ch. 23 Management of patients with Chest and Lower Respiratory Tract Disorders (Brunner & Suddarth)

View Set

Chapter 1: Introduction to Statistics and Research Design

View Set

Clinical Microscopy - Chemical tests

View Set

SYD 3020 final exam study questions and quiz answers

View Set